Prove that $f'(0)$ exists and $f'(0) = b/(a - 1)$











up vote
5
down vote

favorite
2













Problem:

If $f(x)$ is continous at $x=0$, and $limlimits_{xto 0} dfrac{f(ax)-f(x)}{x}=b$, $a, b$ are constants and $|a|>1$, prove that $f'(0)$ exists and $f'(0)=dfrac{b}{a-1}$.




This approach is definitely wrong:



begin{align}
b&=lim_{xto 0} frac{f(ax)-f(x)}{x}\
&=lim_{xto 0} frac{f(ax)-f(0)-(f(x)-f(0))}{x}\
&=af'(0)-f'(0)\
&=(a-1)f'(0)
end{align}



I will show you a case why this approach is wrong:




[f(x)= begin{cases}
1,&xneq0\
0,&x=0
end{cases}]
$lim_{xto0}dfrac{f(3x)-f(x)}{x}=lim_{xto0} dfrac{1-1}{x}=0$

but $lim_{xto0}dfrac{f(3x)}{x}=infty$,$lim_{xto0}dfrac{f(x)}{x}=infty$




Does anyone know how to prove it? Thanks in advance!










share|cite|improve this question
























  • Do you have to also assume that $f'(0)$ exists or does not follow from the other hypotheses?
    – Jonas Meyer
    Jul 19 '16 at 8:41










  • the proof of the existence of $f'(0)$ is actually the problem is asking for
    – Spaceship222
    Jul 19 '16 at 8:47












  • Your result holds even if $|a| < 1$. See update to my answer.
    – Paramanand Singh
    Jul 19 '16 at 12:14










  • Your counter-example violates the continuity hypothesis of $f$ at $x=0$, doesn't it?
    – BusyAnt
    Jul 19 '16 at 12:35










  • @BusyAnt yes it does.I am just not convinced by the approach using the continuity at $x=0$
    – Spaceship222
    Jul 19 '16 at 12:43















up vote
5
down vote

favorite
2













Problem:

If $f(x)$ is continous at $x=0$, and $limlimits_{xto 0} dfrac{f(ax)-f(x)}{x}=b$, $a, b$ are constants and $|a|>1$, prove that $f'(0)$ exists and $f'(0)=dfrac{b}{a-1}$.




This approach is definitely wrong:



begin{align}
b&=lim_{xto 0} frac{f(ax)-f(x)}{x}\
&=lim_{xto 0} frac{f(ax)-f(0)-(f(x)-f(0))}{x}\
&=af'(0)-f'(0)\
&=(a-1)f'(0)
end{align}



I will show you a case why this approach is wrong:




[f(x)= begin{cases}
1,&xneq0\
0,&x=0
end{cases}]
$lim_{xto0}dfrac{f(3x)-f(x)}{x}=lim_{xto0} dfrac{1-1}{x}=0$

but $lim_{xto0}dfrac{f(3x)}{x}=infty$,$lim_{xto0}dfrac{f(x)}{x}=infty$




Does anyone know how to prove it? Thanks in advance!










share|cite|improve this question
























  • Do you have to also assume that $f'(0)$ exists or does not follow from the other hypotheses?
    – Jonas Meyer
    Jul 19 '16 at 8:41










  • the proof of the existence of $f'(0)$ is actually the problem is asking for
    – Spaceship222
    Jul 19 '16 at 8:47












  • Your result holds even if $|a| < 1$. See update to my answer.
    – Paramanand Singh
    Jul 19 '16 at 12:14










  • Your counter-example violates the continuity hypothesis of $f$ at $x=0$, doesn't it?
    – BusyAnt
    Jul 19 '16 at 12:35










  • @BusyAnt yes it does.I am just not convinced by the approach using the continuity at $x=0$
    – Spaceship222
    Jul 19 '16 at 12:43













up vote
5
down vote

favorite
2









up vote
5
down vote

favorite
2






2






Problem:

If $f(x)$ is continous at $x=0$, and $limlimits_{xto 0} dfrac{f(ax)-f(x)}{x}=b$, $a, b$ are constants and $|a|>1$, prove that $f'(0)$ exists and $f'(0)=dfrac{b}{a-1}$.




This approach is definitely wrong:



begin{align}
b&=lim_{xto 0} frac{f(ax)-f(x)}{x}\
&=lim_{xto 0} frac{f(ax)-f(0)-(f(x)-f(0))}{x}\
&=af'(0)-f'(0)\
&=(a-1)f'(0)
end{align}



I will show you a case why this approach is wrong:




[f(x)= begin{cases}
1,&xneq0\
0,&x=0
end{cases}]
$lim_{xto0}dfrac{f(3x)-f(x)}{x}=lim_{xto0} dfrac{1-1}{x}=0$

but $lim_{xto0}dfrac{f(3x)}{x}=infty$,$lim_{xto0}dfrac{f(x)}{x}=infty$




Does anyone know how to prove it? Thanks in advance!










share|cite|improve this question
















Problem:

If $f(x)$ is continous at $x=0$, and $limlimits_{xto 0} dfrac{f(ax)-f(x)}{x}=b$, $a, b$ are constants and $|a|>1$, prove that $f'(0)$ exists and $f'(0)=dfrac{b}{a-1}$.




This approach is definitely wrong:



begin{align}
b&=lim_{xto 0} frac{f(ax)-f(x)}{x}\
&=lim_{xto 0} frac{f(ax)-f(0)-(f(x)-f(0))}{x}\
&=af'(0)-f'(0)\
&=(a-1)f'(0)
end{align}



I will show you a case why this approach is wrong:




[f(x)= begin{cases}
1,&xneq0\
0,&x=0
end{cases}]
$lim_{xto0}dfrac{f(3x)-f(x)}{x}=lim_{xto0} dfrac{1-1}{x}=0$

but $lim_{xto0}dfrac{f(3x)}{x}=infty$,$lim_{xto0}dfrac{f(x)}{x}=infty$




Does anyone know how to prove it? Thanks in advance!







calculus derivatives






share|cite|improve this question















share|cite|improve this question













share|cite|improve this question




share|cite|improve this question








edited Jul 19 '16 at 12:02









Paramanand Singh

48.7k555156




48.7k555156










asked Jul 19 '16 at 8:37









Spaceship222

5517




5517












  • Do you have to also assume that $f'(0)$ exists or does not follow from the other hypotheses?
    – Jonas Meyer
    Jul 19 '16 at 8:41










  • the proof of the existence of $f'(0)$ is actually the problem is asking for
    – Spaceship222
    Jul 19 '16 at 8:47












  • Your result holds even if $|a| < 1$. See update to my answer.
    – Paramanand Singh
    Jul 19 '16 at 12:14










  • Your counter-example violates the continuity hypothesis of $f$ at $x=0$, doesn't it?
    – BusyAnt
    Jul 19 '16 at 12:35










  • @BusyAnt yes it does.I am just not convinced by the approach using the continuity at $x=0$
    – Spaceship222
    Jul 19 '16 at 12:43


















  • Do you have to also assume that $f'(0)$ exists or does not follow from the other hypotheses?
    – Jonas Meyer
    Jul 19 '16 at 8:41










  • the proof of the existence of $f'(0)$ is actually the problem is asking for
    – Spaceship222
    Jul 19 '16 at 8:47












  • Your result holds even if $|a| < 1$. See update to my answer.
    – Paramanand Singh
    Jul 19 '16 at 12:14










  • Your counter-example violates the continuity hypothesis of $f$ at $x=0$, doesn't it?
    – BusyAnt
    Jul 19 '16 at 12:35










  • @BusyAnt yes it does.I am just not convinced by the approach using the continuity at $x=0$
    – Spaceship222
    Jul 19 '16 at 12:43
















Do you have to also assume that $f'(0)$ exists or does not follow from the other hypotheses?
– Jonas Meyer
Jul 19 '16 at 8:41




Do you have to also assume that $f'(0)$ exists or does not follow from the other hypotheses?
– Jonas Meyer
Jul 19 '16 at 8:41












the proof of the existence of $f'(0)$ is actually the problem is asking for
– Spaceship222
Jul 19 '16 at 8:47






the proof of the existence of $f'(0)$ is actually the problem is asking for
– Spaceship222
Jul 19 '16 at 8:47














Your result holds even if $|a| < 1$. See update to my answer.
– Paramanand Singh
Jul 19 '16 at 12:14




Your result holds even if $|a| < 1$. See update to my answer.
– Paramanand Singh
Jul 19 '16 at 12:14












Your counter-example violates the continuity hypothesis of $f$ at $x=0$, doesn't it?
– BusyAnt
Jul 19 '16 at 12:35




Your counter-example violates the continuity hypothesis of $f$ at $x=0$, doesn't it?
– BusyAnt
Jul 19 '16 at 12:35












@BusyAnt yes it does.I am just not convinced by the approach using the continuity at $x=0$
– Spaceship222
Jul 19 '16 at 12:43




@BusyAnt yes it does.I am just not convinced by the approach using the continuity at $x=0$
– Spaceship222
Jul 19 '16 at 12:43










3 Answers
3






active

oldest

votes

















up vote
4
down vote



accepted










This is a tricky question and the solution is somewhat non-obvious. We know that $$lim_{x to 0}frac{f(ax) - f(x)}{x} = b$$ and hence $$f(ax) - f(x) = bx + xg(x)$$ where $g(x) to 0$ as $x to 0$. Replacing $x$ by $x/a$ we get $$f(x) - f(x/a) = bx/a + (x/a)g(x/a)$$ Replacing $x$ by $x/a^{k - 1}$ we get $$f(x/a^{k - 1}) - f(x/a^{k}) = bx/a^{k} + (x/a^{k})g(x/a^{k})$$ Adding such equations for $k = 1, 2, ldots, n$ we get $$f(x) - f(x/a^{n}) = bxsum_{k = 1}^{n}frac{1}{a^{k}} + xsum_{k = 1}^{n}frac{g(x/a^{k})}{a^{k}}$$ Letting $n to infty$ and using sum of infinite GP (remember it converges because $|a| > 1$) and noting that $f$ is continuous at $x = 0$, we get $$f(x) - f(0) = frac{bx}{a - 1} + xsum_{k = 1}^{infty}frac{g(x/a^{k})}{a^{k}}$$ Dividing by $x$ and letting $x to 0$ we get $$f'(0) = lim_{x to 0}frac{f(x) - f(0)}{x} = frac{b}{a - 1} + lim_{x to 0}sum_{k = 1}^{infty}frac{g(x/a^{k})}{a^{k}}$$



The sum $$sum_{k = 1}^{infty}frac{g(x/a^{k})}{a^{k}}$$ tends to $0$ as $x to 0$ because $g(x) to 0$. The proof is not difficult but perhaps not too obvious. Here is one way to do it. Since $g(x)to 0$ as $x to 0$, it follows that for any $epsilon > 0$ there is a $delta > 0$ such that $|g(x)| < epsilon$ for all $x$ with $0 <|x| < delta$. Since $|a| > 1$ it follows that $|x/a^{k}| < delta$ if $|x| < delta$ and therefore $|g(x/a^{k})| < epsilon$. Thus if $0 < |x| < delta$ we have $$left|sum_{k = 1}^{infty}frac{g(x/a^{k})}{a^{k}}right| < sum_{k = 1}^{infty}frac{epsilon}{|a|^{k}} = frac{epsilon}{|a| - 1}$$ and thus the sum tends to $0$ as $x to 0$.



Hence $f'(0) = b/(a - 1)$.





BTW the result in question holds even if $0 < |a| < 1$. Let $c = 1/a$ so that $|c| > 1$. Now we have $$lim_{x to 0}frac{f(ax) - f(x)}{x} = b$$ implies that $$lim_{t to 0}frac{f(ct) - f(t)}{t} = -bc$$ (just put $ax = t$). Hence by what we have proved above it follows that $$f'(0) = frac{-bc}{c - 1} = frac{b}{a - 1}$$ Note that if $a = 1$ then $b = 0$ trivially and we can't say anything about $f'(0)$. And if $a = -1$ then $f(x) = |x|$ provides a counter-example. If $a = 0$ then the result holds trivially by definition of derivative. Hence the result in question holds if and only if $|a| neq 1$.






share|cite|improve this answer























  • How "adding such equation" do you get the left side you did? Do you mean when adding over $;k;$ from $;1;$ to $;n;$? Then, you left $;ntoinfty;$ , but why would $;xto0implies;$ the second summand in the right tends to zero? Even if the right sum converges for all $;x>R;$ , for some $;R>0;$, how it depends on $;x;$ could makea difference.
    – DonAntonio
    Jul 19 '16 at 9:24












  • Yes adding over $k = 1$ to $n$. The infinite sum on right tends to $0$ as $x to 0$. I have kept it as exercise for reader. But It appears I need to prove it. Wait for my updated answer.
    – Paramanand Singh
    Jul 19 '16 at 9:27










  • @DonAntonio: see my updated answer.
    – Paramanand Singh
    Jul 19 '16 at 9:32










  • Thank you. Yet I think this is way too convoluted for an answer, and it may be this exercise is way before infinite series is studied...
    – DonAntonio
    Jul 19 '16 at 9:32








  • 1




    @DonAntonio: The other answer only shows that if $f'(0)$ exists then it must be $b/(a - 1)$. But it does not show why $f'(0)$ exists. One of the downvotes for that answer is mine. In mathematics, correctness is more important than anything else.
    – Paramanand Singh
    Jul 19 '16 at 9:38


















up vote
0
down vote













In this quickly closed question the case $a=2$ is considered, which allows the following simpler solution:



Define $g(x):=f(x)- bx-f(0)$. Then $g$ is continuous at $0$, $g(0)=0$, and $$lim_{xto0}{g(2x)-g(x)over x}=0 .$$
We have to prove that $g'(0)=lim_{xto0}{g(x)over x}=0$.



Let an $epsilon>0$ be given. Then there is a $delta>0$ such that $|g(2t)-g(t)|leqepsilon |t|$ for $0<tleqdelta$. Assume $|x|leqdelta$. Then for each $Nin{mathbb N}$ one has
$$g(x)=sum_{k=1}^Nbigl(g(x/2^{k-1})-g(x/2^k)bigr)+g(x/2^N) ,$$
and therefore
$$bigl|g(x)bigr|leqsum_{k=1}^Nepsilon,{|x|over 2^k} +g(x/2^N)leqepsilon|x|+g(x/2^N) .$$
Since $Nin{mathbb N}$ is arbitrary we in fact have $bigl|g(x)bigr|leq epsilon|x|$, or $left|{g(x)over x}right|leqepsilon$, and this for all $xin>]0,delta]$.






share|cite|improve this answer




























    up vote
    -1
    down vote













    Hint: You're very close.



    Write the expression as $$afrac{f(ax)-f(0)}{ax}-frac{f(x)-f(0)}{x}$$ Note that $xto 0$ if and only if $axto 0$ (since $aneq 0$).



    Can you see it from this?






    share|cite|improve this answer



















    • 2




      $lim_{xto0}frac{f(0)-f(x)}{x}$ is not the same as $lim_{xto0}frac{f(0)-f(ax)}{ax}$ if we don't know the existence of $f'(0)$,so we can't put it together times $(a-1)$
      – Spaceship222
      Jul 19 '16 at 9:02






    • 1




      @MPW I'm rewriting this since I think I understand your comment above better now though it still is pretty messy (the limits are minus the usual one).
      – DonAntonio
      Jul 19 '16 at 9:03






    • 1




      I think, after making some order, both in the above answer and, in particular, in my mind, that this answer is correct: we can write$$b=lim_{xto0}frac{f(ax)-f(x)}x=lim_{xto0}left[afrac{f(ax)-f(0)}{ax}-frac{f(x)-f(0)}xright]=lim_{tto0}(a-1)frac{f(t)-t(0)}t$$because when $;xto0;$ both limits (without the constant $;a;$) within the parentheses are the same, whether it exists or not, because $;f;$ is given continuous at zero and thus it is the same to take $;lim f(x);$ or $;lim f(ax);$ when $xto0$. The rightmost expression, compared to the left side, answers all +1
      – DonAntonio
      Jul 19 '16 at 9:10








    • 1




      I agree here with @Spaceship222: You must prove existence of $f'(0)$ by other means. see my answer. Your answer as it stands is incorrect.
      – Paramanand Singh
      Jul 19 '16 at 9:20








    • 2




      @DonAntonio: Let $F(x) = 1/x$ and $a > 0$. Then both the limits $lim_{x to 0^{+}}F(x)$ and $lim_{x to 0^{+}}F(ax)$ don't exist and yet $$lim_{x to 0^{+}}aF(ax) - F(x) = 0$$ so one should be very careful about the conditions under which laws of algebra of limits work.
      – Paramanand Singh
      Jul 19 '16 at 10:02













    Your Answer





    StackExchange.ifUsing("editor", function () {
    return StackExchange.using("mathjaxEditing", function () {
    StackExchange.MarkdownEditor.creationCallbacks.add(function (editor, postfix) {
    StackExchange.mathjaxEditing.prepareWmdForMathJax(editor, postfix, [["$", "$"], ["\\(","\\)"]]);
    });
    });
    }, "mathjax-editing");

    StackExchange.ready(function() {
    var channelOptions = {
    tags: "".split(" "),
    id: "69"
    };
    initTagRenderer("".split(" "), "".split(" "), channelOptions);

    StackExchange.using("externalEditor", function() {
    // Have to fire editor after snippets, if snippets enabled
    if (StackExchange.settings.snippets.snippetsEnabled) {
    StackExchange.using("snippets", function() {
    createEditor();
    });
    }
    else {
    createEditor();
    }
    });

    function createEditor() {
    StackExchange.prepareEditor({
    heartbeatType: 'answer',
    convertImagesToLinks: true,
    noModals: true,
    showLowRepImageUploadWarning: true,
    reputationToPostImages: 10,
    bindNavPrevention: true,
    postfix: "",
    imageUploader: {
    brandingHtml: "Powered by u003ca class="icon-imgur-white" href="https://imgur.com/"u003eu003c/au003e",
    contentPolicyHtml: "User contributions licensed under u003ca href="https://creativecommons.org/licenses/by-sa/3.0/"u003ecc by-sa 3.0 with attribution requiredu003c/au003e u003ca href="https://stackoverflow.com/legal/content-policy"u003e(content policy)u003c/au003e",
    allowUrls: true
    },
    noCode: true, onDemand: true,
    discardSelector: ".discard-answer"
    ,immediatelyShowMarkdownHelp:true
    });


    }
    });














    draft saved

    draft discarded


















    StackExchange.ready(
    function () {
    StackExchange.openid.initPostLogin('.new-post-login', 'https%3a%2f%2fmath.stackexchange.com%2fquestions%2f1864058%2fprove-that-f0-exists-and-f0-b-a-1%23new-answer', 'question_page');
    }
    );

    Post as a guest















    Required, but never shown

























    3 Answers
    3






    active

    oldest

    votes








    3 Answers
    3






    active

    oldest

    votes









    active

    oldest

    votes






    active

    oldest

    votes








    up vote
    4
    down vote



    accepted










    This is a tricky question and the solution is somewhat non-obvious. We know that $$lim_{x to 0}frac{f(ax) - f(x)}{x} = b$$ and hence $$f(ax) - f(x) = bx + xg(x)$$ where $g(x) to 0$ as $x to 0$. Replacing $x$ by $x/a$ we get $$f(x) - f(x/a) = bx/a + (x/a)g(x/a)$$ Replacing $x$ by $x/a^{k - 1}$ we get $$f(x/a^{k - 1}) - f(x/a^{k}) = bx/a^{k} + (x/a^{k})g(x/a^{k})$$ Adding such equations for $k = 1, 2, ldots, n$ we get $$f(x) - f(x/a^{n}) = bxsum_{k = 1}^{n}frac{1}{a^{k}} + xsum_{k = 1}^{n}frac{g(x/a^{k})}{a^{k}}$$ Letting $n to infty$ and using sum of infinite GP (remember it converges because $|a| > 1$) and noting that $f$ is continuous at $x = 0$, we get $$f(x) - f(0) = frac{bx}{a - 1} + xsum_{k = 1}^{infty}frac{g(x/a^{k})}{a^{k}}$$ Dividing by $x$ and letting $x to 0$ we get $$f'(0) = lim_{x to 0}frac{f(x) - f(0)}{x} = frac{b}{a - 1} + lim_{x to 0}sum_{k = 1}^{infty}frac{g(x/a^{k})}{a^{k}}$$



    The sum $$sum_{k = 1}^{infty}frac{g(x/a^{k})}{a^{k}}$$ tends to $0$ as $x to 0$ because $g(x) to 0$. The proof is not difficult but perhaps not too obvious. Here is one way to do it. Since $g(x)to 0$ as $x to 0$, it follows that for any $epsilon > 0$ there is a $delta > 0$ such that $|g(x)| < epsilon$ for all $x$ with $0 <|x| < delta$. Since $|a| > 1$ it follows that $|x/a^{k}| < delta$ if $|x| < delta$ and therefore $|g(x/a^{k})| < epsilon$. Thus if $0 < |x| < delta$ we have $$left|sum_{k = 1}^{infty}frac{g(x/a^{k})}{a^{k}}right| < sum_{k = 1}^{infty}frac{epsilon}{|a|^{k}} = frac{epsilon}{|a| - 1}$$ and thus the sum tends to $0$ as $x to 0$.



    Hence $f'(0) = b/(a - 1)$.





    BTW the result in question holds even if $0 < |a| < 1$. Let $c = 1/a$ so that $|c| > 1$. Now we have $$lim_{x to 0}frac{f(ax) - f(x)}{x} = b$$ implies that $$lim_{t to 0}frac{f(ct) - f(t)}{t} = -bc$$ (just put $ax = t$). Hence by what we have proved above it follows that $$f'(0) = frac{-bc}{c - 1} = frac{b}{a - 1}$$ Note that if $a = 1$ then $b = 0$ trivially and we can't say anything about $f'(0)$. And if $a = -1$ then $f(x) = |x|$ provides a counter-example. If $a = 0$ then the result holds trivially by definition of derivative. Hence the result in question holds if and only if $|a| neq 1$.






    share|cite|improve this answer























    • How "adding such equation" do you get the left side you did? Do you mean when adding over $;k;$ from $;1;$ to $;n;$? Then, you left $;ntoinfty;$ , but why would $;xto0implies;$ the second summand in the right tends to zero? Even if the right sum converges for all $;x>R;$ , for some $;R>0;$, how it depends on $;x;$ could makea difference.
      – DonAntonio
      Jul 19 '16 at 9:24












    • Yes adding over $k = 1$ to $n$. The infinite sum on right tends to $0$ as $x to 0$. I have kept it as exercise for reader. But It appears I need to prove it. Wait for my updated answer.
      – Paramanand Singh
      Jul 19 '16 at 9:27










    • @DonAntonio: see my updated answer.
      – Paramanand Singh
      Jul 19 '16 at 9:32










    • Thank you. Yet I think this is way too convoluted for an answer, and it may be this exercise is way before infinite series is studied...
      – DonAntonio
      Jul 19 '16 at 9:32








    • 1




      @DonAntonio: The other answer only shows that if $f'(0)$ exists then it must be $b/(a - 1)$. But it does not show why $f'(0)$ exists. One of the downvotes for that answer is mine. In mathematics, correctness is more important than anything else.
      – Paramanand Singh
      Jul 19 '16 at 9:38















    up vote
    4
    down vote



    accepted










    This is a tricky question and the solution is somewhat non-obvious. We know that $$lim_{x to 0}frac{f(ax) - f(x)}{x} = b$$ and hence $$f(ax) - f(x) = bx + xg(x)$$ where $g(x) to 0$ as $x to 0$. Replacing $x$ by $x/a$ we get $$f(x) - f(x/a) = bx/a + (x/a)g(x/a)$$ Replacing $x$ by $x/a^{k - 1}$ we get $$f(x/a^{k - 1}) - f(x/a^{k}) = bx/a^{k} + (x/a^{k})g(x/a^{k})$$ Adding such equations for $k = 1, 2, ldots, n$ we get $$f(x) - f(x/a^{n}) = bxsum_{k = 1}^{n}frac{1}{a^{k}} + xsum_{k = 1}^{n}frac{g(x/a^{k})}{a^{k}}$$ Letting $n to infty$ and using sum of infinite GP (remember it converges because $|a| > 1$) and noting that $f$ is continuous at $x = 0$, we get $$f(x) - f(0) = frac{bx}{a - 1} + xsum_{k = 1}^{infty}frac{g(x/a^{k})}{a^{k}}$$ Dividing by $x$ and letting $x to 0$ we get $$f'(0) = lim_{x to 0}frac{f(x) - f(0)}{x} = frac{b}{a - 1} + lim_{x to 0}sum_{k = 1}^{infty}frac{g(x/a^{k})}{a^{k}}$$



    The sum $$sum_{k = 1}^{infty}frac{g(x/a^{k})}{a^{k}}$$ tends to $0$ as $x to 0$ because $g(x) to 0$. The proof is not difficult but perhaps not too obvious. Here is one way to do it. Since $g(x)to 0$ as $x to 0$, it follows that for any $epsilon > 0$ there is a $delta > 0$ such that $|g(x)| < epsilon$ for all $x$ with $0 <|x| < delta$. Since $|a| > 1$ it follows that $|x/a^{k}| < delta$ if $|x| < delta$ and therefore $|g(x/a^{k})| < epsilon$. Thus if $0 < |x| < delta$ we have $$left|sum_{k = 1}^{infty}frac{g(x/a^{k})}{a^{k}}right| < sum_{k = 1}^{infty}frac{epsilon}{|a|^{k}} = frac{epsilon}{|a| - 1}$$ and thus the sum tends to $0$ as $x to 0$.



    Hence $f'(0) = b/(a - 1)$.





    BTW the result in question holds even if $0 < |a| < 1$. Let $c = 1/a$ so that $|c| > 1$. Now we have $$lim_{x to 0}frac{f(ax) - f(x)}{x} = b$$ implies that $$lim_{t to 0}frac{f(ct) - f(t)}{t} = -bc$$ (just put $ax = t$). Hence by what we have proved above it follows that $$f'(0) = frac{-bc}{c - 1} = frac{b}{a - 1}$$ Note that if $a = 1$ then $b = 0$ trivially and we can't say anything about $f'(0)$. And if $a = -1$ then $f(x) = |x|$ provides a counter-example. If $a = 0$ then the result holds trivially by definition of derivative. Hence the result in question holds if and only if $|a| neq 1$.






    share|cite|improve this answer























    • How "adding such equation" do you get the left side you did? Do you mean when adding over $;k;$ from $;1;$ to $;n;$? Then, you left $;ntoinfty;$ , but why would $;xto0implies;$ the second summand in the right tends to zero? Even if the right sum converges for all $;x>R;$ , for some $;R>0;$, how it depends on $;x;$ could makea difference.
      – DonAntonio
      Jul 19 '16 at 9:24












    • Yes adding over $k = 1$ to $n$. The infinite sum on right tends to $0$ as $x to 0$. I have kept it as exercise for reader. But It appears I need to prove it. Wait for my updated answer.
      – Paramanand Singh
      Jul 19 '16 at 9:27










    • @DonAntonio: see my updated answer.
      – Paramanand Singh
      Jul 19 '16 at 9:32










    • Thank you. Yet I think this is way too convoluted for an answer, and it may be this exercise is way before infinite series is studied...
      – DonAntonio
      Jul 19 '16 at 9:32








    • 1




      @DonAntonio: The other answer only shows that if $f'(0)$ exists then it must be $b/(a - 1)$. But it does not show why $f'(0)$ exists. One of the downvotes for that answer is mine. In mathematics, correctness is more important than anything else.
      – Paramanand Singh
      Jul 19 '16 at 9:38













    up vote
    4
    down vote



    accepted







    up vote
    4
    down vote



    accepted






    This is a tricky question and the solution is somewhat non-obvious. We know that $$lim_{x to 0}frac{f(ax) - f(x)}{x} = b$$ and hence $$f(ax) - f(x) = bx + xg(x)$$ where $g(x) to 0$ as $x to 0$. Replacing $x$ by $x/a$ we get $$f(x) - f(x/a) = bx/a + (x/a)g(x/a)$$ Replacing $x$ by $x/a^{k - 1}$ we get $$f(x/a^{k - 1}) - f(x/a^{k}) = bx/a^{k} + (x/a^{k})g(x/a^{k})$$ Adding such equations for $k = 1, 2, ldots, n$ we get $$f(x) - f(x/a^{n}) = bxsum_{k = 1}^{n}frac{1}{a^{k}} + xsum_{k = 1}^{n}frac{g(x/a^{k})}{a^{k}}$$ Letting $n to infty$ and using sum of infinite GP (remember it converges because $|a| > 1$) and noting that $f$ is continuous at $x = 0$, we get $$f(x) - f(0) = frac{bx}{a - 1} + xsum_{k = 1}^{infty}frac{g(x/a^{k})}{a^{k}}$$ Dividing by $x$ and letting $x to 0$ we get $$f'(0) = lim_{x to 0}frac{f(x) - f(0)}{x} = frac{b}{a - 1} + lim_{x to 0}sum_{k = 1}^{infty}frac{g(x/a^{k})}{a^{k}}$$



    The sum $$sum_{k = 1}^{infty}frac{g(x/a^{k})}{a^{k}}$$ tends to $0$ as $x to 0$ because $g(x) to 0$. The proof is not difficult but perhaps not too obvious. Here is one way to do it. Since $g(x)to 0$ as $x to 0$, it follows that for any $epsilon > 0$ there is a $delta > 0$ such that $|g(x)| < epsilon$ for all $x$ with $0 <|x| < delta$. Since $|a| > 1$ it follows that $|x/a^{k}| < delta$ if $|x| < delta$ and therefore $|g(x/a^{k})| < epsilon$. Thus if $0 < |x| < delta$ we have $$left|sum_{k = 1}^{infty}frac{g(x/a^{k})}{a^{k}}right| < sum_{k = 1}^{infty}frac{epsilon}{|a|^{k}} = frac{epsilon}{|a| - 1}$$ and thus the sum tends to $0$ as $x to 0$.



    Hence $f'(0) = b/(a - 1)$.





    BTW the result in question holds even if $0 < |a| < 1$. Let $c = 1/a$ so that $|c| > 1$. Now we have $$lim_{x to 0}frac{f(ax) - f(x)}{x} = b$$ implies that $$lim_{t to 0}frac{f(ct) - f(t)}{t} = -bc$$ (just put $ax = t$). Hence by what we have proved above it follows that $$f'(0) = frac{-bc}{c - 1} = frac{b}{a - 1}$$ Note that if $a = 1$ then $b = 0$ trivially and we can't say anything about $f'(0)$. And if $a = -1$ then $f(x) = |x|$ provides a counter-example. If $a = 0$ then the result holds trivially by definition of derivative. Hence the result in question holds if and only if $|a| neq 1$.






    share|cite|improve this answer














    This is a tricky question and the solution is somewhat non-obvious. We know that $$lim_{x to 0}frac{f(ax) - f(x)}{x} = b$$ and hence $$f(ax) - f(x) = bx + xg(x)$$ where $g(x) to 0$ as $x to 0$. Replacing $x$ by $x/a$ we get $$f(x) - f(x/a) = bx/a + (x/a)g(x/a)$$ Replacing $x$ by $x/a^{k - 1}$ we get $$f(x/a^{k - 1}) - f(x/a^{k}) = bx/a^{k} + (x/a^{k})g(x/a^{k})$$ Adding such equations for $k = 1, 2, ldots, n$ we get $$f(x) - f(x/a^{n}) = bxsum_{k = 1}^{n}frac{1}{a^{k}} + xsum_{k = 1}^{n}frac{g(x/a^{k})}{a^{k}}$$ Letting $n to infty$ and using sum of infinite GP (remember it converges because $|a| > 1$) and noting that $f$ is continuous at $x = 0$, we get $$f(x) - f(0) = frac{bx}{a - 1} + xsum_{k = 1}^{infty}frac{g(x/a^{k})}{a^{k}}$$ Dividing by $x$ and letting $x to 0$ we get $$f'(0) = lim_{x to 0}frac{f(x) - f(0)}{x} = frac{b}{a - 1} + lim_{x to 0}sum_{k = 1}^{infty}frac{g(x/a^{k})}{a^{k}}$$



    The sum $$sum_{k = 1}^{infty}frac{g(x/a^{k})}{a^{k}}$$ tends to $0$ as $x to 0$ because $g(x) to 0$. The proof is not difficult but perhaps not too obvious. Here is one way to do it. Since $g(x)to 0$ as $x to 0$, it follows that for any $epsilon > 0$ there is a $delta > 0$ such that $|g(x)| < epsilon$ for all $x$ with $0 <|x| < delta$. Since $|a| > 1$ it follows that $|x/a^{k}| < delta$ if $|x| < delta$ and therefore $|g(x/a^{k})| < epsilon$. Thus if $0 < |x| < delta$ we have $$left|sum_{k = 1}^{infty}frac{g(x/a^{k})}{a^{k}}right| < sum_{k = 1}^{infty}frac{epsilon}{|a|^{k}} = frac{epsilon}{|a| - 1}$$ and thus the sum tends to $0$ as $x to 0$.



    Hence $f'(0) = b/(a - 1)$.





    BTW the result in question holds even if $0 < |a| < 1$. Let $c = 1/a$ so that $|c| > 1$. Now we have $$lim_{x to 0}frac{f(ax) - f(x)}{x} = b$$ implies that $$lim_{t to 0}frac{f(ct) - f(t)}{t} = -bc$$ (just put $ax = t$). Hence by what we have proved above it follows that $$f'(0) = frac{-bc}{c - 1} = frac{b}{a - 1}$$ Note that if $a = 1$ then $b = 0$ trivially and we can't say anything about $f'(0)$. And if $a = -1$ then $f(x) = |x|$ provides a counter-example. If $a = 0$ then the result holds trivially by definition of derivative. Hence the result in question holds if and only if $|a| neq 1$.







    share|cite|improve this answer














    share|cite|improve this answer



    share|cite|improve this answer








    edited Jul 25 '16 at 8:09

























    answered Jul 19 '16 at 9:18









    Paramanand Singh

    48.7k555156




    48.7k555156












    • How "adding such equation" do you get the left side you did? Do you mean when adding over $;k;$ from $;1;$ to $;n;$? Then, you left $;ntoinfty;$ , but why would $;xto0implies;$ the second summand in the right tends to zero? Even if the right sum converges for all $;x>R;$ , for some $;R>0;$, how it depends on $;x;$ could makea difference.
      – DonAntonio
      Jul 19 '16 at 9:24












    • Yes adding over $k = 1$ to $n$. The infinite sum on right tends to $0$ as $x to 0$. I have kept it as exercise for reader. But It appears I need to prove it. Wait for my updated answer.
      – Paramanand Singh
      Jul 19 '16 at 9:27










    • @DonAntonio: see my updated answer.
      – Paramanand Singh
      Jul 19 '16 at 9:32










    • Thank you. Yet I think this is way too convoluted for an answer, and it may be this exercise is way before infinite series is studied...
      – DonAntonio
      Jul 19 '16 at 9:32








    • 1




      @DonAntonio: The other answer only shows that if $f'(0)$ exists then it must be $b/(a - 1)$. But it does not show why $f'(0)$ exists. One of the downvotes for that answer is mine. In mathematics, correctness is more important than anything else.
      – Paramanand Singh
      Jul 19 '16 at 9:38


















    • How "adding such equation" do you get the left side you did? Do you mean when adding over $;k;$ from $;1;$ to $;n;$? Then, you left $;ntoinfty;$ , but why would $;xto0implies;$ the second summand in the right tends to zero? Even if the right sum converges for all $;x>R;$ , for some $;R>0;$, how it depends on $;x;$ could makea difference.
      – DonAntonio
      Jul 19 '16 at 9:24












    • Yes adding over $k = 1$ to $n$. The infinite sum on right tends to $0$ as $x to 0$. I have kept it as exercise for reader. But It appears I need to prove it. Wait for my updated answer.
      – Paramanand Singh
      Jul 19 '16 at 9:27










    • @DonAntonio: see my updated answer.
      – Paramanand Singh
      Jul 19 '16 at 9:32










    • Thank you. Yet I think this is way too convoluted for an answer, and it may be this exercise is way before infinite series is studied...
      – DonAntonio
      Jul 19 '16 at 9:32








    • 1




      @DonAntonio: The other answer only shows that if $f'(0)$ exists then it must be $b/(a - 1)$. But it does not show why $f'(0)$ exists. One of the downvotes for that answer is mine. In mathematics, correctness is more important than anything else.
      – Paramanand Singh
      Jul 19 '16 at 9:38
















    How "adding such equation" do you get the left side you did? Do you mean when adding over $;k;$ from $;1;$ to $;n;$? Then, you left $;ntoinfty;$ , but why would $;xto0implies;$ the second summand in the right tends to zero? Even if the right sum converges for all $;x>R;$ , for some $;R>0;$, how it depends on $;x;$ could makea difference.
    – DonAntonio
    Jul 19 '16 at 9:24






    How "adding such equation" do you get the left side you did? Do you mean when adding over $;k;$ from $;1;$ to $;n;$? Then, you left $;ntoinfty;$ , but why would $;xto0implies;$ the second summand in the right tends to zero? Even if the right sum converges for all $;x>R;$ , for some $;R>0;$, how it depends on $;x;$ could makea difference.
    – DonAntonio
    Jul 19 '16 at 9:24














    Yes adding over $k = 1$ to $n$. The infinite sum on right tends to $0$ as $x to 0$. I have kept it as exercise for reader. But It appears I need to prove it. Wait for my updated answer.
    – Paramanand Singh
    Jul 19 '16 at 9:27




    Yes adding over $k = 1$ to $n$. The infinite sum on right tends to $0$ as $x to 0$. I have kept it as exercise for reader. But It appears I need to prove it. Wait for my updated answer.
    – Paramanand Singh
    Jul 19 '16 at 9:27












    @DonAntonio: see my updated answer.
    – Paramanand Singh
    Jul 19 '16 at 9:32




    @DonAntonio: see my updated answer.
    – Paramanand Singh
    Jul 19 '16 at 9:32












    Thank you. Yet I think this is way too convoluted for an answer, and it may be this exercise is way before infinite series is studied...
    – DonAntonio
    Jul 19 '16 at 9:32






    Thank you. Yet I think this is way too convoluted for an answer, and it may be this exercise is way before infinite series is studied...
    – DonAntonio
    Jul 19 '16 at 9:32






    1




    1




    @DonAntonio: The other answer only shows that if $f'(0)$ exists then it must be $b/(a - 1)$. But it does not show why $f'(0)$ exists. One of the downvotes for that answer is mine. In mathematics, correctness is more important than anything else.
    – Paramanand Singh
    Jul 19 '16 at 9:38




    @DonAntonio: The other answer only shows that if $f'(0)$ exists then it must be $b/(a - 1)$. But it does not show why $f'(0)$ exists. One of the downvotes for that answer is mine. In mathematics, correctness is more important than anything else.
    – Paramanand Singh
    Jul 19 '16 at 9:38










    up vote
    0
    down vote













    In this quickly closed question the case $a=2$ is considered, which allows the following simpler solution:



    Define $g(x):=f(x)- bx-f(0)$. Then $g$ is continuous at $0$, $g(0)=0$, and $$lim_{xto0}{g(2x)-g(x)over x}=0 .$$
    We have to prove that $g'(0)=lim_{xto0}{g(x)over x}=0$.



    Let an $epsilon>0$ be given. Then there is a $delta>0$ such that $|g(2t)-g(t)|leqepsilon |t|$ for $0<tleqdelta$. Assume $|x|leqdelta$. Then for each $Nin{mathbb N}$ one has
    $$g(x)=sum_{k=1}^Nbigl(g(x/2^{k-1})-g(x/2^k)bigr)+g(x/2^N) ,$$
    and therefore
    $$bigl|g(x)bigr|leqsum_{k=1}^Nepsilon,{|x|over 2^k} +g(x/2^N)leqepsilon|x|+g(x/2^N) .$$
    Since $Nin{mathbb N}$ is arbitrary we in fact have $bigl|g(x)bigr|leq epsilon|x|$, or $left|{g(x)over x}right|leqepsilon$, and this for all $xin>]0,delta]$.






    share|cite|improve this answer

























      up vote
      0
      down vote













      In this quickly closed question the case $a=2$ is considered, which allows the following simpler solution:



      Define $g(x):=f(x)- bx-f(0)$. Then $g$ is continuous at $0$, $g(0)=0$, and $$lim_{xto0}{g(2x)-g(x)over x}=0 .$$
      We have to prove that $g'(0)=lim_{xto0}{g(x)over x}=0$.



      Let an $epsilon>0$ be given. Then there is a $delta>0$ such that $|g(2t)-g(t)|leqepsilon |t|$ for $0<tleqdelta$. Assume $|x|leqdelta$. Then for each $Nin{mathbb N}$ one has
      $$g(x)=sum_{k=1}^Nbigl(g(x/2^{k-1})-g(x/2^k)bigr)+g(x/2^N) ,$$
      and therefore
      $$bigl|g(x)bigr|leqsum_{k=1}^Nepsilon,{|x|over 2^k} +g(x/2^N)leqepsilon|x|+g(x/2^N) .$$
      Since $Nin{mathbb N}$ is arbitrary we in fact have $bigl|g(x)bigr|leq epsilon|x|$, or $left|{g(x)over x}right|leqepsilon$, and this for all $xin>]0,delta]$.






      share|cite|improve this answer























        up vote
        0
        down vote










        up vote
        0
        down vote









        In this quickly closed question the case $a=2$ is considered, which allows the following simpler solution:



        Define $g(x):=f(x)- bx-f(0)$. Then $g$ is continuous at $0$, $g(0)=0$, and $$lim_{xto0}{g(2x)-g(x)over x}=0 .$$
        We have to prove that $g'(0)=lim_{xto0}{g(x)over x}=0$.



        Let an $epsilon>0$ be given. Then there is a $delta>0$ such that $|g(2t)-g(t)|leqepsilon |t|$ for $0<tleqdelta$. Assume $|x|leqdelta$. Then for each $Nin{mathbb N}$ one has
        $$g(x)=sum_{k=1}^Nbigl(g(x/2^{k-1})-g(x/2^k)bigr)+g(x/2^N) ,$$
        and therefore
        $$bigl|g(x)bigr|leqsum_{k=1}^Nepsilon,{|x|over 2^k} +g(x/2^N)leqepsilon|x|+g(x/2^N) .$$
        Since $Nin{mathbb N}$ is arbitrary we in fact have $bigl|g(x)bigr|leq epsilon|x|$, or $left|{g(x)over x}right|leqepsilon$, and this for all $xin>]0,delta]$.






        share|cite|improve this answer












        In this quickly closed question the case $a=2$ is considered, which allows the following simpler solution:



        Define $g(x):=f(x)- bx-f(0)$. Then $g$ is continuous at $0$, $g(0)=0$, and $$lim_{xto0}{g(2x)-g(x)over x}=0 .$$
        We have to prove that $g'(0)=lim_{xto0}{g(x)over x}=0$.



        Let an $epsilon>0$ be given. Then there is a $delta>0$ such that $|g(2t)-g(t)|leqepsilon |t|$ for $0<tleqdelta$. Assume $|x|leqdelta$. Then for each $Nin{mathbb N}$ one has
        $$g(x)=sum_{k=1}^Nbigl(g(x/2^{k-1})-g(x/2^k)bigr)+g(x/2^N) ,$$
        and therefore
        $$bigl|g(x)bigr|leqsum_{k=1}^Nepsilon,{|x|over 2^k} +g(x/2^N)leqepsilon|x|+g(x/2^N) .$$
        Since $Nin{mathbb N}$ is arbitrary we in fact have $bigl|g(x)bigr|leq epsilon|x|$, or $left|{g(x)over x}right|leqepsilon$, and this for all $xin>]0,delta]$.







        share|cite|improve this answer












        share|cite|improve this answer



        share|cite|improve this answer










        answered Nov 24 at 15:17









        Christian Blatter

        171k7111325




        171k7111325






















            up vote
            -1
            down vote













            Hint: You're very close.



            Write the expression as $$afrac{f(ax)-f(0)}{ax}-frac{f(x)-f(0)}{x}$$ Note that $xto 0$ if and only if $axto 0$ (since $aneq 0$).



            Can you see it from this?






            share|cite|improve this answer



















            • 2




              $lim_{xto0}frac{f(0)-f(x)}{x}$ is not the same as $lim_{xto0}frac{f(0)-f(ax)}{ax}$ if we don't know the existence of $f'(0)$,so we can't put it together times $(a-1)$
              – Spaceship222
              Jul 19 '16 at 9:02






            • 1




              @MPW I'm rewriting this since I think I understand your comment above better now though it still is pretty messy (the limits are minus the usual one).
              – DonAntonio
              Jul 19 '16 at 9:03






            • 1




              I think, after making some order, both in the above answer and, in particular, in my mind, that this answer is correct: we can write$$b=lim_{xto0}frac{f(ax)-f(x)}x=lim_{xto0}left[afrac{f(ax)-f(0)}{ax}-frac{f(x)-f(0)}xright]=lim_{tto0}(a-1)frac{f(t)-t(0)}t$$because when $;xto0;$ both limits (without the constant $;a;$) within the parentheses are the same, whether it exists or not, because $;f;$ is given continuous at zero and thus it is the same to take $;lim f(x);$ or $;lim f(ax);$ when $xto0$. The rightmost expression, compared to the left side, answers all +1
              – DonAntonio
              Jul 19 '16 at 9:10








            • 1




              I agree here with @Spaceship222: You must prove existence of $f'(0)$ by other means. see my answer. Your answer as it stands is incorrect.
              – Paramanand Singh
              Jul 19 '16 at 9:20








            • 2




              @DonAntonio: Let $F(x) = 1/x$ and $a > 0$. Then both the limits $lim_{x to 0^{+}}F(x)$ and $lim_{x to 0^{+}}F(ax)$ don't exist and yet $$lim_{x to 0^{+}}aF(ax) - F(x) = 0$$ so one should be very careful about the conditions under which laws of algebra of limits work.
              – Paramanand Singh
              Jul 19 '16 at 10:02

















            up vote
            -1
            down vote













            Hint: You're very close.



            Write the expression as $$afrac{f(ax)-f(0)}{ax}-frac{f(x)-f(0)}{x}$$ Note that $xto 0$ if and only if $axto 0$ (since $aneq 0$).



            Can you see it from this?






            share|cite|improve this answer



















            • 2




              $lim_{xto0}frac{f(0)-f(x)}{x}$ is not the same as $lim_{xto0}frac{f(0)-f(ax)}{ax}$ if we don't know the existence of $f'(0)$,so we can't put it together times $(a-1)$
              – Spaceship222
              Jul 19 '16 at 9:02






            • 1




              @MPW I'm rewriting this since I think I understand your comment above better now though it still is pretty messy (the limits are minus the usual one).
              – DonAntonio
              Jul 19 '16 at 9:03






            • 1




              I think, after making some order, both in the above answer and, in particular, in my mind, that this answer is correct: we can write$$b=lim_{xto0}frac{f(ax)-f(x)}x=lim_{xto0}left[afrac{f(ax)-f(0)}{ax}-frac{f(x)-f(0)}xright]=lim_{tto0}(a-1)frac{f(t)-t(0)}t$$because when $;xto0;$ both limits (without the constant $;a;$) within the parentheses are the same, whether it exists or not, because $;f;$ is given continuous at zero and thus it is the same to take $;lim f(x);$ or $;lim f(ax);$ when $xto0$. The rightmost expression, compared to the left side, answers all +1
              – DonAntonio
              Jul 19 '16 at 9:10








            • 1




              I agree here with @Spaceship222: You must prove existence of $f'(0)$ by other means. see my answer. Your answer as it stands is incorrect.
              – Paramanand Singh
              Jul 19 '16 at 9:20








            • 2




              @DonAntonio: Let $F(x) = 1/x$ and $a > 0$. Then both the limits $lim_{x to 0^{+}}F(x)$ and $lim_{x to 0^{+}}F(ax)$ don't exist and yet $$lim_{x to 0^{+}}aF(ax) - F(x) = 0$$ so one should be very careful about the conditions under which laws of algebra of limits work.
              – Paramanand Singh
              Jul 19 '16 at 10:02















            up vote
            -1
            down vote










            up vote
            -1
            down vote









            Hint: You're very close.



            Write the expression as $$afrac{f(ax)-f(0)}{ax}-frac{f(x)-f(0)}{x}$$ Note that $xto 0$ if and only if $axto 0$ (since $aneq 0$).



            Can you see it from this?






            share|cite|improve this answer














            Hint: You're very close.



            Write the expression as $$afrac{f(ax)-f(0)}{ax}-frac{f(x)-f(0)}{x}$$ Note that $xto 0$ if and only if $axto 0$ (since $aneq 0$).



            Can you see it from this?







            share|cite|improve this answer














            share|cite|improve this answer



            share|cite|improve this answer








            edited Jul 19 '16 at 12:31

























            answered Jul 19 '16 at 8:48









            MPW

            29.8k11956




            29.8k11956








            • 2




              $lim_{xto0}frac{f(0)-f(x)}{x}$ is not the same as $lim_{xto0}frac{f(0)-f(ax)}{ax}$ if we don't know the existence of $f'(0)$,so we can't put it together times $(a-1)$
              – Spaceship222
              Jul 19 '16 at 9:02






            • 1




              @MPW I'm rewriting this since I think I understand your comment above better now though it still is pretty messy (the limits are minus the usual one).
              – DonAntonio
              Jul 19 '16 at 9:03






            • 1




              I think, after making some order, both in the above answer and, in particular, in my mind, that this answer is correct: we can write$$b=lim_{xto0}frac{f(ax)-f(x)}x=lim_{xto0}left[afrac{f(ax)-f(0)}{ax}-frac{f(x)-f(0)}xright]=lim_{tto0}(a-1)frac{f(t)-t(0)}t$$because when $;xto0;$ both limits (without the constant $;a;$) within the parentheses are the same, whether it exists or not, because $;f;$ is given continuous at zero and thus it is the same to take $;lim f(x);$ or $;lim f(ax);$ when $xto0$. The rightmost expression, compared to the left side, answers all +1
              – DonAntonio
              Jul 19 '16 at 9:10








            • 1




              I agree here with @Spaceship222: You must prove existence of $f'(0)$ by other means. see my answer. Your answer as it stands is incorrect.
              – Paramanand Singh
              Jul 19 '16 at 9:20








            • 2




              @DonAntonio: Let $F(x) = 1/x$ and $a > 0$. Then both the limits $lim_{x to 0^{+}}F(x)$ and $lim_{x to 0^{+}}F(ax)$ don't exist and yet $$lim_{x to 0^{+}}aF(ax) - F(x) = 0$$ so one should be very careful about the conditions under which laws of algebra of limits work.
              – Paramanand Singh
              Jul 19 '16 at 10:02
















            • 2




              $lim_{xto0}frac{f(0)-f(x)}{x}$ is not the same as $lim_{xto0}frac{f(0)-f(ax)}{ax}$ if we don't know the existence of $f'(0)$,so we can't put it together times $(a-1)$
              – Spaceship222
              Jul 19 '16 at 9:02






            • 1




              @MPW I'm rewriting this since I think I understand your comment above better now though it still is pretty messy (the limits are minus the usual one).
              – DonAntonio
              Jul 19 '16 at 9:03






            • 1




              I think, after making some order, both in the above answer and, in particular, in my mind, that this answer is correct: we can write$$b=lim_{xto0}frac{f(ax)-f(x)}x=lim_{xto0}left[afrac{f(ax)-f(0)}{ax}-frac{f(x)-f(0)}xright]=lim_{tto0}(a-1)frac{f(t)-t(0)}t$$because when $;xto0;$ both limits (without the constant $;a;$) within the parentheses are the same, whether it exists or not, because $;f;$ is given continuous at zero and thus it is the same to take $;lim f(x);$ or $;lim f(ax);$ when $xto0$. The rightmost expression, compared to the left side, answers all +1
              – DonAntonio
              Jul 19 '16 at 9:10








            • 1




              I agree here with @Spaceship222: You must prove existence of $f'(0)$ by other means. see my answer. Your answer as it stands is incorrect.
              – Paramanand Singh
              Jul 19 '16 at 9:20








            • 2




              @DonAntonio: Let $F(x) = 1/x$ and $a > 0$. Then both the limits $lim_{x to 0^{+}}F(x)$ and $lim_{x to 0^{+}}F(ax)$ don't exist and yet $$lim_{x to 0^{+}}aF(ax) - F(x) = 0$$ so one should be very careful about the conditions under which laws of algebra of limits work.
              – Paramanand Singh
              Jul 19 '16 at 10:02










            2




            2




            $lim_{xto0}frac{f(0)-f(x)}{x}$ is not the same as $lim_{xto0}frac{f(0)-f(ax)}{ax}$ if we don't know the existence of $f'(0)$,so we can't put it together times $(a-1)$
            – Spaceship222
            Jul 19 '16 at 9:02




            $lim_{xto0}frac{f(0)-f(x)}{x}$ is not the same as $lim_{xto0}frac{f(0)-f(ax)}{ax}$ if we don't know the existence of $f'(0)$,so we can't put it together times $(a-1)$
            – Spaceship222
            Jul 19 '16 at 9:02




            1




            1




            @MPW I'm rewriting this since I think I understand your comment above better now though it still is pretty messy (the limits are minus the usual one).
            – DonAntonio
            Jul 19 '16 at 9:03




            @MPW I'm rewriting this since I think I understand your comment above better now though it still is pretty messy (the limits are minus the usual one).
            – DonAntonio
            Jul 19 '16 at 9:03




            1




            1




            I think, after making some order, both in the above answer and, in particular, in my mind, that this answer is correct: we can write$$b=lim_{xto0}frac{f(ax)-f(x)}x=lim_{xto0}left[afrac{f(ax)-f(0)}{ax}-frac{f(x)-f(0)}xright]=lim_{tto0}(a-1)frac{f(t)-t(0)}t$$because when $;xto0;$ both limits (without the constant $;a;$) within the parentheses are the same, whether it exists or not, because $;f;$ is given continuous at zero and thus it is the same to take $;lim f(x);$ or $;lim f(ax);$ when $xto0$. The rightmost expression, compared to the left side, answers all +1
            – DonAntonio
            Jul 19 '16 at 9:10






            I think, after making some order, both in the above answer and, in particular, in my mind, that this answer is correct: we can write$$b=lim_{xto0}frac{f(ax)-f(x)}x=lim_{xto0}left[afrac{f(ax)-f(0)}{ax}-frac{f(x)-f(0)}xright]=lim_{tto0}(a-1)frac{f(t)-t(0)}t$$because when $;xto0;$ both limits (without the constant $;a;$) within the parentheses are the same, whether it exists or not, because $;f;$ is given continuous at zero and thus it is the same to take $;lim f(x);$ or $;lim f(ax);$ when $xto0$. The rightmost expression, compared to the left side, answers all +1
            – DonAntonio
            Jul 19 '16 at 9:10






            1




            1




            I agree here with @Spaceship222: You must prove existence of $f'(0)$ by other means. see my answer. Your answer as it stands is incorrect.
            – Paramanand Singh
            Jul 19 '16 at 9:20






            I agree here with @Spaceship222: You must prove existence of $f'(0)$ by other means. see my answer. Your answer as it stands is incorrect.
            – Paramanand Singh
            Jul 19 '16 at 9:20






            2




            2




            @DonAntonio: Let $F(x) = 1/x$ and $a > 0$. Then both the limits $lim_{x to 0^{+}}F(x)$ and $lim_{x to 0^{+}}F(ax)$ don't exist and yet $$lim_{x to 0^{+}}aF(ax) - F(x) = 0$$ so one should be very careful about the conditions under which laws of algebra of limits work.
            – Paramanand Singh
            Jul 19 '16 at 10:02






            @DonAntonio: Let $F(x) = 1/x$ and $a > 0$. Then both the limits $lim_{x to 0^{+}}F(x)$ and $lim_{x to 0^{+}}F(ax)$ don't exist and yet $$lim_{x to 0^{+}}aF(ax) - F(x) = 0$$ so one should be very careful about the conditions under which laws of algebra of limits work.
            – Paramanand Singh
            Jul 19 '16 at 10:02




















            draft saved

            draft discarded




















































            Thanks for contributing an answer to Mathematics Stack Exchange!


            • Please be sure to answer the question. Provide details and share your research!

            But avoid



            • Asking for help, clarification, or responding to other answers.

            • Making statements based on opinion; back them up with references or personal experience.


            Use MathJax to format equations. MathJax reference.


            To learn more, see our tips on writing great answers.





            Some of your past answers have not been well-received, and you're in danger of being blocked from answering.


            Please pay close attention to the following guidance:


            • Please be sure to answer the question. Provide details and share your research!

            But avoid



            • Asking for help, clarification, or responding to other answers.

            • Making statements based on opinion; back them up with references or personal experience.


            To learn more, see our tips on writing great answers.




            draft saved


            draft discarded














            StackExchange.ready(
            function () {
            StackExchange.openid.initPostLogin('.new-post-login', 'https%3a%2f%2fmath.stackexchange.com%2fquestions%2f1864058%2fprove-that-f0-exists-and-f0-b-a-1%23new-answer', 'question_page');
            }
            );

            Post as a guest















            Required, but never shown





















































            Required, but never shown














            Required, but never shown












            Required, but never shown







            Required, but never shown

































            Required, but never shown














            Required, but never shown












            Required, but never shown







            Required, but never shown







            Popular posts from this blog

            Le Mesnil-Réaume

            Ida-Boy-Ed-Garten

            web3.py web3.isConnected() returns false always